Partitioning the positive integers into finitely many arithmetic progressions

The name of the pictureThe name of the pictureThe name of the pictureClash Royale CLAN TAG#URR8PPP












2












$begingroup$


From Bóna's A Walk through Combinatorics:




Prove or disprove that if we partition the positive integers into finitely many arithmetic progressions then there will be at least one arithmetic progression whose difference and initial term are equal.




A variant of this problems asks to prove that there will be at least one arithmetic progression whose difference divides the initial term, which has an elementary proof. I suspect the same arithmetic progression will have the initial term equal to the difference. However, I cannot prove it.










share|cite|improve this question











$endgroup$







  • 2




    $begingroup$
    What is the origin of this question? Usually if you ask to "prove" something you should have some strong evidence that the proof exists.
    $endgroup$
    – Fedor Petrov
    Feb 25 at 14:30






  • 1




    $begingroup$
    Sorry I should’ve asked for counter example also. I have edited the question now. The source is the variant mentioned below the main question which is an exercise from “A Walk Through Combinatorics”.
    $endgroup$
    – John
    Feb 25 at 15:27










  • $begingroup$
    Counter example where there are an infinite number of infinite arithmetic progressions and NO progression has the difference equal to the initial term. for k from 1 to inf S(k) = 2^k * n + 2 ^ (k-1) for n from 0 to inf S(1) is odd integers; S(2) is 2, 6, 10 - twice an odd integer; and continues for powers of 2. In all cases the initial term is the difference / 2.
    $endgroup$
    – Judah Greenblatt
    Feb 25 at 21:43















2












$begingroup$


From Bóna's A Walk through Combinatorics:




Prove or disprove that if we partition the positive integers into finitely many arithmetic progressions then there will be at least one arithmetic progression whose difference and initial term are equal.




A variant of this problems asks to prove that there will be at least one arithmetic progression whose difference divides the initial term, which has an elementary proof. I suspect the same arithmetic progression will have the initial term equal to the difference. However, I cannot prove it.










share|cite|improve this question











$endgroup$







  • 2




    $begingroup$
    What is the origin of this question? Usually if you ask to "prove" something you should have some strong evidence that the proof exists.
    $endgroup$
    – Fedor Petrov
    Feb 25 at 14:30






  • 1




    $begingroup$
    Sorry I should’ve asked for counter example also. I have edited the question now. The source is the variant mentioned below the main question which is an exercise from “A Walk Through Combinatorics”.
    $endgroup$
    – John
    Feb 25 at 15:27










  • $begingroup$
    Counter example where there are an infinite number of infinite arithmetic progressions and NO progression has the difference equal to the initial term. for k from 1 to inf S(k) = 2^k * n + 2 ^ (k-1) for n from 0 to inf S(1) is odd integers; S(2) is 2, 6, 10 - twice an odd integer; and continues for powers of 2. In all cases the initial term is the difference / 2.
    $endgroup$
    – Judah Greenblatt
    Feb 25 at 21:43













2












2








2





$begingroup$


From Bóna's A Walk through Combinatorics:




Prove or disprove that if we partition the positive integers into finitely many arithmetic progressions then there will be at least one arithmetic progression whose difference and initial term are equal.




A variant of this problems asks to prove that there will be at least one arithmetic progression whose difference divides the initial term, which has an elementary proof. I suspect the same arithmetic progression will have the initial term equal to the difference. However, I cannot prove it.










share|cite|improve this question











$endgroup$




From Bóna's A Walk through Combinatorics:




Prove or disprove that if we partition the positive integers into finitely many arithmetic progressions then there will be at least one arithmetic progression whose difference and initial term are equal.




A variant of this problems asks to prove that there will be at least one arithmetic progression whose difference divides the initial term, which has an elementary proof. I suspect the same arithmetic progression will have the initial term equal to the difference. However, I cannot prove it.







arithmetic-progression elementary-proofs






share|cite|improve this question















share|cite|improve this question













share|cite|improve this question




share|cite|improve this question








edited Feb 25 at 21:02









Rodrigo de Azevedo

1,8422719




1,8422719










asked Feb 25 at 13:28









JohnJohn

194




194







  • 2




    $begingroup$
    What is the origin of this question? Usually if you ask to "prove" something you should have some strong evidence that the proof exists.
    $endgroup$
    – Fedor Petrov
    Feb 25 at 14:30






  • 1




    $begingroup$
    Sorry I should’ve asked for counter example also. I have edited the question now. The source is the variant mentioned below the main question which is an exercise from “A Walk Through Combinatorics”.
    $endgroup$
    – John
    Feb 25 at 15:27










  • $begingroup$
    Counter example where there are an infinite number of infinite arithmetic progressions and NO progression has the difference equal to the initial term. for k from 1 to inf S(k) = 2^k * n + 2 ^ (k-1) for n from 0 to inf S(1) is odd integers; S(2) is 2, 6, 10 - twice an odd integer; and continues for powers of 2. In all cases the initial term is the difference / 2.
    $endgroup$
    – Judah Greenblatt
    Feb 25 at 21:43












  • 2




    $begingroup$
    What is the origin of this question? Usually if you ask to "prove" something you should have some strong evidence that the proof exists.
    $endgroup$
    – Fedor Petrov
    Feb 25 at 14:30






  • 1




    $begingroup$
    Sorry I should’ve asked for counter example also. I have edited the question now. The source is the variant mentioned below the main question which is an exercise from “A Walk Through Combinatorics”.
    $endgroup$
    – John
    Feb 25 at 15:27










  • $begingroup$
    Counter example where there are an infinite number of infinite arithmetic progressions and NO progression has the difference equal to the initial term. for k from 1 to inf S(k) = 2^k * n + 2 ^ (k-1) for n from 0 to inf S(1) is odd integers; S(2) is 2, 6, 10 - twice an odd integer; and continues for powers of 2. In all cases the initial term is the difference / 2.
    $endgroup$
    – Judah Greenblatt
    Feb 25 at 21:43







2




2




$begingroup$
What is the origin of this question? Usually if you ask to "prove" something you should have some strong evidence that the proof exists.
$endgroup$
– Fedor Petrov
Feb 25 at 14:30




$begingroup$
What is the origin of this question? Usually if you ask to "prove" something you should have some strong evidence that the proof exists.
$endgroup$
– Fedor Petrov
Feb 25 at 14:30




1




1




$begingroup$
Sorry I should’ve asked for counter example also. I have edited the question now. The source is the variant mentioned below the main question which is an exercise from “A Walk Through Combinatorics”.
$endgroup$
– John
Feb 25 at 15:27




$begingroup$
Sorry I should’ve asked for counter example also. I have edited the question now. The source is the variant mentioned below the main question which is an exercise from “A Walk Through Combinatorics”.
$endgroup$
– John
Feb 25 at 15:27












$begingroup$
Counter example where there are an infinite number of infinite arithmetic progressions and NO progression has the difference equal to the initial term. for k from 1 to inf S(k) = 2^k * n + 2 ^ (k-1) for n from 0 to inf S(1) is odd integers; S(2) is 2, 6, 10 - twice an odd integer; and continues for powers of 2. In all cases the initial term is the difference / 2.
$endgroup$
– Judah Greenblatt
Feb 25 at 21:43




$begingroup$
Counter example where there are an infinite number of infinite arithmetic progressions and NO progression has the difference equal to the initial term. for k from 1 to inf S(k) = 2^k * n + 2 ^ (k-1) for n from 0 to inf S(1) is odd integers; S(2) is 2, 6, 10 - twice an odd integer; and continues for powers of 2. In all cases the initial term is the difference / 2.
$endgroup$
– Judah Greenblatt
Feb 25 at 21:43










1 Answer
1






active

oldest

votes


















7












$begingroup$

Any progression (if they are all infinite, of course, but otherwise the statement is clearly wrong) should have its initial term $a$ not greater than the difference $d$. Indeed, if $a>d$, and $a-d$ is covered by another progression $P$ with difference $d_1$, then $a-d+dd_1$ is covered twice --- a contradiction. Therefore if $a$ is divisible by $d$, we must have $a=d$. And such $a$ exists as you noted in OP (a short proof: take a progression which contains the product of all differences).






share|cite|improve this answer











$endgroup$












    Your Answer





    StackExchange.ifUsing("editor", function ()
    return StackExchange.using("mathjaxEditing", function ()
    StackExchange.MarkdownEditor.creationCallbacks.add(function (editor, postfix)
    StackExchange.mathjaxEditing.prepareWmdForMathJax(editor, postfix, [["$", "$"], ["\\(","\\)"]]);
    );
    );
    , "mathjax-editing");

    StackExchange.ready(function()
    var channelOptions =
    tags: "".split(" "),
    id: "504"
    ;
    initTagRenderer("".split(" "), "".split(" "), channelOptions);

    StackExchange.using("externalEditor", function()
    // Have to fire editor after snippets, if snippets enabled
    if (StackExchange.settings.snippets.snippetsEnabled)
    StackExchange.using("snippets", function()
    createEditor();
    );

    else
    createEditor();

    );

    function createEditor()
    StackExchange.prepareEditor(
    heartbeatType: 'answer',
    autoActivateHeartbeat: false,
    convertImagesToLinks: true,
    noModals: true,
    showLowRepImageUploadWarning: true,
    reputationToPostImages: 10,
    bindNavPrevention: true,
    postfix: "",
    imageUploader:
    brandingHtml: "Powered by u003ca class="icon-imgur-white" href="https://imgur.com/"u003eu003c/au003e",
    contentPolicyHtml: "User contributions licensed under u003ca href="https://creativecommons.org/licenses/by-sa/3.0/"u003ecc by-sa 3.0 with attribution requiredu003c/au003e u003ca href="https://stackoverflow.com/legal/content-policy"u003e(content policy)u003c/au003e",
    allowUrls: true
    ,
    noCode: true, onDemand: true,
    discardSelector: ".discard-answer"
    ,immediatelyShowMarkdownHelp:true
    );



    );













    draft saved

    draft discarded


















    StackExchange.ready(
    function ()
    StackExchange.openid.initPostLogin('.new-post-login', 'https%3a%2f%2fmathoverflow.net%2fquestions%2f324088%2fpartitioning-the-positive-integers-into-finitely-many-arithmetic-progressions%23new-answer', 'question_page');

    );

    Post as a guest















    Required, but never shown

























    1 Answer
    1






    active

    oldest

    votes








    1 Answer
    1






    active

    oldest

    votes









    active

    oldest

    votes






    active

    oldest

    votes









    7












    $begingroup$

    Any progression (if they are all infinite, of course, but otherwise the statement is clearly wrong) should have its initial term $a$ not greater than the difference $d$. Indeed, if $a>d$, and $a-d$ is covered by another progression $P$ with difference $d_1$, then $a-d+dd_1$ is covered twice --- a contradiction. Therefore if $a$ is divisible by $d$, we must have $a=d$. And such $a$ exists as you noted in OP (a short proof: take a progression which contains the product of all differences).






    share|cite|improve this answer











    $endgroup$

















      7












      $begingroup$

      Any progression (if they are all infinite, of course, but otherwise the statement is clearly wrong) should have its initial term $a$ not greater than the difference $d$. Indeed, if $a>d$, and $a-d$ is covered by another progression $P$ with difference $d_1$, then $a-d+dd_1$ is covered twice --- a contradiction. Therefore if $a$ is divisible by $d$, we must have $a=d$. And such $a$ exists as you noted in OP (a short proof: take a progression which contains the product of all differences).






      share|cite|improve this answer











      $endgroup$















        7












        7








        7





        $begingroup$

        Any progression (if they are all infinite, of course, but otherwise the statement is clearly wrong) should have its initial term $a$ not greater than the difference $d$. Indeed, if $a>d$, and $a-d$ is covered by another progression $P$ with difference $d_1$, then $a-d+dd_1$ is covered twice --- a contradiction. Therefore if $a$ is divisible by $d$, we must have $a=d$. And such $a$ exists as you noted in OP (a short proof: take a progression which contains the product of all differences).






        share|cite|improve this answer











        $endgroup$



        Any progression (if they are all infinite, of course, but otherwise the statement is clearly wrong) should have its initial term $a$ not greater than the difference $d$. Indeed, if $a>d$, and $a-d$ is covered by another progression $P$ with difference $d_1$, then $a-d+dd_1$ is covered twice --- a contradiction. Therefore if $a$ is divisible by $d$, we must have $a=d$. And such $a$ exists as you noted in OP (a short proof: take a progression which contains the product of all differences).







        share|cite|improve this answer














        share|cite|improve this answer



        share|cite|improve this answer








        edited Feb 25 at 18:51

























        answered Feb 25 at 15:43









        Fedor PetrovFedor Petrov

        51.1k6118235




        51.1k6118235



























            draft saved

            draft discarded
















































            Thanks for contributing an answer to MathOverflow!


            • Please be sure to answer the question. Provide details and share your research!

            But avoid


            • Asking for help, clarification, or responding to other answers.

            • Making statements based on opinion; back them up with references or personal experience.

            Use MathJax to format equations. MathJax reference.


            To learn more, see our tips on writing great answers.




            draft saved


            draft discarded














            StackExchange.ready(
            function ()
            StackExchange.openid.initPostLogin('.new-post-login', 'https%3a%2f%2fmathoverflow.net%2fquestions%2f324088%2fpartitioning-the-positive-integers-into-finitely-many-arithmetic-progressions%23new-answer', 'question_page');

            );

            Post as a guest















            Required, but never shown





















































            Required, but never shown














            Required, but never shown












            Required, but never shown







            Required, but never shown

































            Required, but never shown














            Required, but never shown












            Required, but never shown







            Required, but never shown






            Popular posts from this blog

            How to check contact read email or not when send email to Individual?

            Bahrain

            Postfix configuration issue with fips on centos 7; mailgun relay